Prove the cross of two open sets is also an open set.












0














$U$ and $V$ are open subsets of $mathbb{R}$. Show that $U×V = {(x,y)mid xin U, yin V }$ is an open subset of $mathbb{R}^{2}$.



I know the definition of an open subset is that $forall (x,y) in U×V, exists delta > 0$ such that $Bleft((x,y), deltaright) subset U×V$.



I just don't know how to use that in this case.










share|cite|improve this question
























  • We use a type of $LaTeX$ here called MathJax. There's an easy-to-find tutorial on it on the meta site.
    – Shaun
    Nov 27 '18 at 2:28












  • Please edit the question accordingly.
    – Shaun
    Nov 27 '18 at 2:29
















0














$U$ and $V$ are open subsets of $mathbb{R}$. Show that $U×V = {(x,y)mid xin U, yin V }$ is an open subset of $mathbb{R}^{2}$.



I know the definition of an open subset is that $forall (x,y) in U×V, exists delta > 0$ such that $Bleft((x,y), deltaright) subset U×V$.



I just don't know how to use that in this case.










share|cite|improve this question
























  • We use a type of $LaTeX$ here called MathJax. There's an easy-to-find tutorial on it on the meta site.
    – Shaun
    Nov 27 '18 at 2:28












  • Please edit the question accordingly.
    – Shaun
    Nov 27 '18 at 2:29














0












0








0







$U$ and $V$ are open subsets of $mathbb{R}$. Show that $U×V = {(x,y)mid xin U, yin V }$ is an open subset of $mathbb{R}^{2}$.



I know the definition of an open subset is that $forall (x,y) in U×V, exists delta > 0$ such that $Bleft((x,y), deltaright) subset U×V$.



I just don't know how to use that in this case.










share|cite|improve this question















$U$ and $V$ are open subsets of $mathbb{R}$. Show that $U×V = {(x,y)mid xin U, yin V }$ is an open subset of $mathbb{R}^{2}$.



I know the definition of an open subset is that $forall (x,y) in U×V, exists delta > 0$ such that $Bleft((x,y), deltaright) subset U×V$.



I just don't know how to use that in this case.







real-analysis metric-spaces






share|cite|improve this question















share|cite|improve this question













share|cite|improve this question




share|cite|improve this question








edited Nov 28 '18 at 2:51

























asked Nov 27 '18 at 2:20









kendal

337




337












  • We use a type of $LaTeX$ here called MathJax. There's an easy-to-find tutorial on it on the meta site.
    – Shaun
    Nov 27 '18 at 2:28












  • Please edit the question accordingly.
    – Shaun
    Nov 27 '18 at 2:29


















  • We use a type of $LaTeX$ here called MathJax. There's an easy-to-find tutorial on it on the meta site.
    – Shaun
    Nov 27 '18 at 2:28












  • Please edit the question accordingly.
    – Shaun
    Nov 27 '18 at 2:29
















We use a type of $LaTeX$ here called MathJax. There's an easy-to-find tutorial on it on the meta site.
– Shaun
Nov 27 '18 at 2:28






We use a type of $LaTeX$ here called MathJax. There's an easy-to-find tutorial on it on the meta site.
– Shaun
Nov 27 '18 at 2:28














Please edit the question accordingly.
– Shaun
Nov 27 '18 at 2:29




Please edit the question accordingly.
– Shaun
Nov 27 '18 at 2:29










1 Answer
1






active

oldest

votes


















2














Let $(x,y)in Utimes V$ be given.



Note that $xin U$ and $yin V$, so there exists $delta_x >0$ and $delta_y>0$ such that $$B(x,delta_x)subseteq U iff text{ and } B(y,delta_y)subseteq V.$$



Note that whenever $ain B(x,delta_x)$, we have $|x-a|<delta_x$. And similarly $|y-b|<delta_y$ whenever $bin B(y,delta_y)$.



Let $delta =min(delta_x,delta_y)$.



Now let $(a,b)in B((x,y),delta)$, then we have $sqrt{(x-a)^2+(y-b)^2}<delta$.



It implies that $|x-a|<deltaleq delta_x$ and that $|y-b|< delta leq delta_y$ which means that $ain U$ and $bin V$, so $(a,b)in Utimes V$.



Therefore, $B((x,y),delta)subseteq Utimes V$ and we are done.






share|cite|improve this answer





















    Your Answer





    StackExchange.ifUsing("editor", function () {
    return StackExchange.using("mathjaxEditing", function () {
    StackExchange.MarkdownEditor.creationCallbacks.add(function (editor, postfix) {
    StackExchange.mathjaxEditing.prepareWmdForMathJax(editor, postfix, [["$", "$"], ["\\(","\\)"]]);
    });
    });
    }, "mathjax-editing");

    StackExchange.ready(function() {
    var channelOptions = {
    tags: "".split(" "),
    id: "69"
    };
    initTagRenderer("".split(" "), "".split(" "), channelOptions);

    StackExchange.using("externalEditor", function() {
    // Have to fire editor after snippets, if snippets enabled
    if (StackExchange.settings.snippets.snippetsEnabled) {
    StackExchange.using("snippets", function() {
    createEditor();
    });
    }
    else {
    createEditor();
    }
    });

    function createEditor() {
    StackExchange.prepareEditor({
    heartbeatType: 'answer',
    autoActivateHeartbeat: false,
    convertImagesToLinks: true,
    noModals: true,
    showLowRepImageUploadWarning: true,
    reputationToPostImages: 10,
    bindNavPrevention: true,
    postfix: "",
    imageUploader: {
    brandingHtml: "Powered by u003ca class="icon-imgur-white" href="https://imgur.com/"u003eu003c/au003e",
    contentPolicyHtml: "User contributions licensed under u003ca href="https://creativecommons.org/licenses/by-sa/3.0/"u003ecc by-sa 3.0 with attribution requiredu003c/au003e u003ca href="https://stackoverflow.com/legal/content-policy"u003e(content policy)u003c/au003e",
    allowUrls: true
    },
    noCode: true, onDemand: true,
    discardSelector: ".discard-answer"
    ,immediatelyShowMarkdownHelp:true
    });


    }
    });














    draft saved

    draft discarded


















    StackExchange.ready(
    function () {
    StackExchange.openid.initPostLogin('.new-post-login', 'https%3a%2f%2fmath.stackexchange.com%2fquestions%2f3015241%2fprove-the-cross-of-two-open-sets-is-also-an-open-set%23new-answer', 'question_page');
    }
    );

    Post as a guest















    Required, but never shown

























    1 Answer
    1






    active

    oldest

    votes








    1 Answer
    1






    active

    oldest

    votes









    active

    oldest

    votes






    active

    oldest

    votes









    2














    Let $(x,y)in Utimes V$ be given.



    Note that $xin U$ and $yin V$, so there exists $delta_x >0$ and $delta_y>0$ such that $$B(x,delta_x)subseteq U iff text{ and } B(y,delta_y)subseteq V.$$



    Note that whenever $ain B(x,delta_x)$, we have $|x-a|<delta_x$. And similarly $|y-b|<delta_y$ whenever $bin B(y,delta_y)$.



    Let $delta =min(delta_x,delta_y)$.



    Now let $(a,b)in B((x,y),delta)$, then we have $sqrt{(x-a)^2+(y-b)^2}<delta$.



    It implies that $|x-a|<deltaleq delta_x$ and that $|y-b|< delta leq delta_y$ which means that $ain U$ and $bin V$, so $(a,b)in Utimes V$.



    Therefore, $B((x,y),delta)subseteq Utimes V$ and we are done.






    share|cite|improve this answer


























      2














      Let $(x,y)in Utimes V$ be given.



      Note that $xin U$ and $yin V$, so there exists $delta_x >0$ and $delta_y>0$ such that $$B(x,delta_x)subseteq U iff text{ and } B(y,delta_y)subseteq V.$$



      Note that whenever $ain B(x,delta_x)$, we have $|x-a|<delta_x$. And similarly $|y-b|<delta_y$ whenever $bin B(y,delta_y)$.



      Let $delta =min(delta_x,delta_y)$.



      Now let $(a,b)in B((x,y),delta)$, then we have $sqrt{(x-a)^2+(y-b)^2}<delta$.



      It implies that $|x-a|<deltaleq delta_x$ and that $|y-b|< delta leq delta_y$ which means that $ain U$ and $bin V$, so $(a,b)in Utimes V$.



      Therefore, $B((x,y),delta)subseteq Utimes V$ and we are done.






      share|cite|improve this answer
























        2












        2








        2






        Let $(x,y)in Utimes V$ be given.



        Note that $xin U$ and $yin V$, so there exists $delta_x >0$ and $delta_y>0$ such that $$B(x,delta_x)subseteq U iff text{ and } B(y,delta_y)subseteq V.$$



        Note that whenever $ain B(x,delta_x)$, we have $|x-a|<delta_x$. And similarly $|y-b|<delta_y$ whenever $bin B(y,delta_y)$.



        Let $delta =min(delta_x,delta_y)$.



        Now let $(a,b)in B((x,y),delta)$, then we have $sqrt{(x-a)^2+(y-b)^2}<delta$.



        It implies that $|x-a|<deltaleq delta_x$ and that $|y-b|< delta leq delta_y$ which means that $ain U$ and $bin V$, so $(a,b)in Utimes V$.



        Therefore, $B((x,y),delta)subseteq Utimes V$ and we are done.






        share|cite|improve this answer












        Let $(x,y)in Utimes V$ be given.



        Note that $xin U$ and $yin V$, so there exists $delta_x >0$ and $delta_y>0$ such that $$B(x,delta_x)subseteq U iff text{ and } B(y,delta_y)subseteq V.$$



        Note that whenever $ain B(x,delta_x)$, we have $|x-a|<delta_x$. And similarly $|y-b|<delta_y$ whenever $bin B(y,delta_y)$.



        Let $delta =min(delta_x,delta_y)$.



        Now let $(a,b)in B((x,y),delta)$, then we have $sqrt{(x-a)^2+(y-b)^2}<delta$.



        It implies that $|x-a|<deltaleq delta_x$ and that $|y-b|< delta leq delta_y$ which means that $ain U$ and $bin V$, so $(a,b)in Utimes V$.



        Therefore, $B((x,y),delta)subseteq Utimes V$ and we are done.







        share|cite|improve this answer












        share|cite|improve this answer



        share|cite|improve this answer










        answered Nov 27 '18 at 2:39









        LeB

        986217




        986217






























            draft saved

            draft discarded




















































            Thanks for contributing an answer to Mathematics Stack Exchange!


            • Please be sure to answer the question. Provide details and share your research!

            But avoid



            • Asking for help, clarification, or responding to other answers.

            • Making statements based on opinion; back them up with references or personal experience.


            Use MathJax to format equations. MathJax reference.


            To learn more, see our tips on writing great answers.





            Some of your past answers have not been well-received, and you're in danger of being blocked from answering.


            Please pay close attention to the following guidance:


            • Please be sure to answer the question. Provide details and share your research!

            But avoid



            • Asking for help, clarification, or responding to other answers.

            • Making statements based on opinion; back them up with references or personal experience.


            To learn more, see our tips on writing great answers.




            draft saved


            draft discarded














            StackExchange.ready(
            function () {
            StackExchange.openid.initPostLogin('.new-post-login', 'https%3a%2f%2fmath.stackexchange.com%2fquestions%2f3015241%2fprove-the-cross-of-two-open-sets-is-also-an-open-set%23new-answer', 'question_page');
            }
            );

            Post as a guest















            Required, but never shown





















































            Required, but never shown














            Required, but never shown












            Required, but never shown







            Required, but never shown

































            Required, but never shown














            Required, but never shown












            Required, but never shown







            Required, but never shown







            Popular posts from this blog

            Quarter-circle Tiles

            build a pushdown automaton that recognizes the reverse language of a given pushdown automaton?

            Mont Emei